Show that the polar line of the exincenter passes through this midpoint

euclidean-geometrygeometryprojective-geometry

We're given a triangle $\triangle ABC$ whose incenter is $I$ and its $A$-exincenter is $I_A$.

enter image description here

Let line $EF$ be the polar of $A$ with respect to the incircle of $\triangle ABC$, let $G = EF \cap BC$, let $D$ be the point of that incircle who is also on $BC$. Show that the polar line of $I_A$ wrt the incircle of $\triangle ABC$ passes through the midpoint $M$ of $GD$.

It is easy to see that the polar of $I_A$ is parallel to $EF$ but I'm struggling with that midpoint. Is there some hidden projection here?

EDIT: they're suggesting me to do length chasing on the comments below.

So: $BD = p-b = \frac{a+c-b}2$
$\frac{x}{x+a} = \frac{c}b \iff GB = \frac{ac}{b-c}$
so $MB = \frac12(\frac{ac}{b-c} + b-p)$

what now?

EDIT2: Ok, now we got the final hint in the comments.

It is clear that $\mathcal H (G,D;B,C)$ which implies that

$$MD^2 = MB \cdot MC$$

Look how interesting: if the mark the intersections of the green polar with the incircle and name them $XY$, then quad $XYBD$ is cyclic as $MX \cdot MY = MD^2 = MB \cdot MC$.

Now I need to justify why $I_A$ is on $(XYBC)$. Which I'm struggling a bit.

Best Answer

Your attempt is on the right track, though somewhat in reverse.

For the sake of completeness, I will post the full solution.

Lemma 1: Given four collinear points $A,C,B,D$ and $X$, the midpoint of $AB$, then $(AB;CD) = -1$ iff $XC\cdot XD = XB^2$.
Proof: Using directed lengths: $$\frac{AC}{AD} = \frac{CB}{BD} \iff \frac{AC}{CB} = \frac{AD}{BD} \iff \frac{AC+CB}{AC+BC} = \frac{AD+BD}{AD+DB}$$ $$\iff XB^2 = \frac{AB^2}{4} = \frac{(AC+BC)(AD+BD)}{4} = \frac{(AB + BC + BC)(AB+BD+BD)}{4} = (XB+BD)(XB+BC) = XD \cdot XC$$

Lemma 2: Let $DEF$ be the contact triangle of $\triangle ABC$ ($D$ on $BC$, $E$ on $AC$ and $F$ on $AB$), and let $T = EF \cap BC$. Then $(TD;BC) = -1$.
Proof: Let $AD \cap \odot(DEF) = L$, then it is well-known that $LEDF$ is a harmonic quadrilateral. Then we must have $(T,DL \cap EF; E,F) = -1$ (lemma 2 and 3 here), and $(TD;BC) \overset{A}{=} (T,DL\cap EF; E,F) = -1$ as desired.

It is well known by the incenter-excenter lemma (see a proof here) that $BI_ACI$ is cyclic with diameter $II_A$. Using the notation in OP, here is another lemma:

Lemma 3: $X,Y$ lie on $\odot(BIC)$.
Proof: Trivial once you notice $\angle IXI_A = \angle IYI_A = 90^\circ$.

It is not hard to see the finish from here:

Let $M'$ be the intersection of the polar of $I_A$ with $BC$. Then by lemma 3, $M'B \cdot M'C = M'X\cdot M'Y = M'D^2$. However, by lemma 1, this means that $M'$ is the midpoint of $D$ and its harmonic conjugate w.r.t. $BC$, which by lemma 2, is $G$.